Last visit was: 26 Apr 2024, 20:04 It is currently 26 Apr 2024, 20:04

Close
GMAT Club Daily Prep
Thank you for using the timer - this advanced tool can estimate your performance and suggest more practice questions. We have subscribed you to Daily Prep Questions via email.

Customized
for You

we will pick new questions that match your level based on your Timer History

Track
Your Progress

every week, we’ll send you an estimated GMAT score based on your performance

Practice
Pays

we will pick new questions that match your level based on your Timer History
Not interested in getting valuable practice questions and articles delivered to your email? No problem, unsubscribe here.
Close
Request Expert Reply
Confirm Cancel
SORT BY:
Date
Tags:
Show Tags
Hide Tags
Manager
Manager
Joined: 08 Mar 2019
Status:students
Posts: 74
Own Kudos [?]: 305 [3]
Given Kudos: 28
GPA: 3.9
WE:Account Management (Accounting)
Send PM
GMAT Tutor
Joined: 24 Jun 2008
Posts: 4128
Own Kudos [?]: 9247 [2]
Given Kudos: 91
 Q51  V47
Send PM
Intern
Intern
Joined: 03 Nov 2018
Posts: 25
Own Kudos [?]: 25 [0]
Given Kudos: 97
Send PM
Senior Manager
Senior Manager
Joined: 22 Nov 2018
Posts: 446
Own Kudos [?]: 492 [0]
Given Kudos: 292
Location: India
GMAT 1: 640 Q45 V35
GMAT 2: 740 Q49 V41
Send PM
Re: Halfway through a 100-shot archery tournament, Chelsea leads by 50 poi [#permalink]
My solution
Max points that can be lost on each shot is 6 points. Lead is 50. So for 8 shots they can afford to not hit bulls eye while opponents hit bullseye. So 42 guarantees victory

Posted from my mobile device
Intern
Intern
Joined: 21 Sep 2018
Posts: 24
Own Kudos [?]: 15 [0]
Given Kudos: 15
Location: India
Concentration: Finance, General Management
GMAT 1: 650 Q47 V31
Send PM
Re: Halfway through a 100-shot archery tournament, Chelsea leads by 50 poi [#permalink]
IanStewart wrote:
She has a 50 point lead, and on the final fifty shots, the person in second place can score at most 500 points (10 points per shot). So as long as she scores more than 450 points on her final fifty shots, she will be certain to win.

If she gets n consecutive bullseyes, scoring 10 points each, she will get 10n points in total. Then on her final 50-n shots, we know she also scores at least 4 points, so she will get at least another 4(50 - n) = 200 - 4n points. So she will score at least 10n + 200 - 4n = 6n + 200 points. This needs to be greater than 450, so

6n + 200 > 450
6n > 250
n > 41 + 2/3

and the smallest possible integer value of n is 42.

Why did you assume that she scored only 4 points on 50-n shots? The question says atleast 4. it could be 4 or 8. Any specific reason to select 4?
User avatar
Non-Human User
Joined: 09 Sep 2013
Posts: 32691
Own Kudos [?]: 822 [0]
Given Kudos: 0
Send PM
Re: Halfway through a 100-shot archery tournament, Chelsea leads by 50 poi [#permalink]
Hello from the GMAT Club BumpBot!

Thanks to another GMAT Club member, I have just discovered this valuable topic, yet it had no discussion for over a year. I am now bumping it up - doing my job. I think you may find it valuable (esp those replies with Kudos).

Want to see all other topics I dig out? Follow me (click follow button on profile). You will receive a summary of all topics I bump in your profile area as well as via email.
GMAT Club Bot
Re: Halfway through a 100-shot archery tournament, Chelsea leads by 50 poi [#permalink]
Moderators:
Math Expert
92948 posts
Senior Moderator - Masters Forum
3137 posts

Powered by phpBB © phpBB Group | Emoji artwork provided by EmojiOne